Help
Could you please explain?
yckim2180 on April 8 at 09:30PM
  • October 2003 LSAT
  • SEC4
  • Q17
1
Reply
Can someone please explain this
Can someone please explain this? I am completely lost
EugeneC on April 8 at 09:29PM
  • October 2003 LSAT
  • SEC4
  • Q17
1
Reply
Why A?
Where is the support for this?
Nishant-Varma on April 8 at 02:36AM
  • October 2003 LSAT
  • SEC4
  • Q12
1
Reply
Why not B?
Can someone please explain?
Jimmywantstogotolawschool on April 8 at 02:34AM
  • October 2003 LSAT
  • SEC4
  • Q6
1
Reply
Answer choice D
Hello, how is choice D the correct answer?
amarachicynthia on April 8 at 02:33AM
  • October 2003 LSAT
  • SEC4
  • Q6
1
Reply
B vs E
At first, I was going to select E. However, then, the lines stating in bringing suit confused me....
hfatima1 on March 31 at 02:15PM
  • October 2003 LSAT
  • SEC4
  • Q2
1
Reply
Why A and not D?
Can you explain how A is the main point?
Nishant-Varma on March 31 at 02:14PM
  • October 2003 LSAT
  • SEC4
  • Q1
1
Reply
Why not B?
Does that not weaken the claim as well?
Nishant-Varma on February 13, 2022
  • October 2003 LSAT
  • SEC4
  • Q20
5
Replies
Why is the correct answer choice D instead of A?
D and A seem similar to me. I had the answer choices narrowed down between the two but I ended up...
Francesca on February 12, 2022
  • October 2003 LSAT
  • SEC4
  • Q14
2
Replies
Please explain
Can you please explain this question?
Steph on February 7, 2022
  • October 2003 LSAT
  • SEC4
  • Q20
6
Replies
Why not E?
I see why A is correct, but I picked E though it was wrong because I thought the reason renewable...
jingjingxiao11111@gmail.com on April 12, 2021
  • October 2003 LSAT
  • SEC4
  • Q15
2
Replies
Why not A
this is the only question I missed in this section, i was really sure that the first paragraph wa...
hfatima1 on October 31, 2020
  • October 2003 LSAT
  • SEC4
  • Q13
1
Reply
Why D
I didn't choose D because of the phrase new practice nothing in the passage indicates that the pr...
Meredith on April 14, 2020
  • October 2003 LSAT
  • SEC4
  • Q4
3
Replies
Victorian philanthropy
The two modern criticisms are the "earlier" and "recent" criticisms, right? And the former was sa...
Lily on April 14, 2020
  • October 2003 LSAT
  • SEC4
  • Q22
2
Replies
Where do you find the answer for this?
Looking for help finding the reasoning.
Tucker-Sutlive on March 17, 2020
  • October 2003 LSAT
  • SEC4
  • Q25
6
Replies
Please explain
I don't understand why E is the correct answer
Steph on February 4, 2020
  • October 2003 LSAT
  • SEC4
  • Q25
3
Replies
Why A?
I'm very confused as to why the correct answer is A. Where in the passage is this supported? Why ...
Meredith on October 29, 2019
  • October 2003 LSAT
  • SEC4
  • Q15
1
Reply
Renewable sources of energy
Whyyy A??? Is it that if they lol at long term and use local involvement in all levels, they too...
Lily on September 30, 2014
  • October 2003 LSAT
  • SEC4
  • Q15
3
Replies